LSAT and Law School Admissions Forum

Get expert LSAT preparation and law school admissions advice from PowerScore Test Preparation.

User avatar
 Dave Killoran
PowerScore Staff
  • PowerScore Staff
  • Posts: 5850
  • Joined: Mar 25, 2011
|
#49158
Complete Question Explanation
(The complete setup for this game can be found here: lsat/viewtopic.php?t=9142)

The correct answer choice is (D)

Again, most of the incorrect answer choices are eliminated by the initial inferences.

Answer choice (A) is incorrect because F and M cannot be prescribed together.

Answer choice (B) is incorrect because F and W cannot be prescribed together.

Answer choice (C) is incorrect because V and N cannot be prescribed together.

Answer choice (D) is the correct answer choice.

Answer choice (E) is incorrect because V and N cannot be prescribed together.

Get the most out of your LSAT Prep Plus subscription.

Analyze and track your performance with our Testing and Analytics Package.